Difference between revisions of "2024 AMC 8 Problems/Problem 19"
m (→Solution) |
(→Problem) |
||
Line 1: | Line 1: | ||
==Problem== | ==Problem== | ||
+ | |||
+ | Jordan owns <math>15</math> pairs of sneakers. Three fifths of the pairs are red and the rest are white. Two thirds of the pairs are high-top and the rest are low-top. The red high-top sneakers make up a fraction of the collection. What is the least possible value of this fraction? | ||
+ | |||
+ | (A) <math>0</math> (B) <math>\frac{1}{5}</math> (C) <math>\frac{4}{15}</math> (D) <math>\frac{1}{3}</math> (E) <math>\frac{2}{5}</math> | ||
+ | |||
==Solution== | ==Solution== | ||
Jordan has <math>10</math> high top sneakers, and <math>6</math> white sneakers. We would want as many white high-top sneakers as possible, so we set <math>6</math> high-top sneakers to be white. Then, we have <math>10-6=4</math> red high-top sneakers, so the answer is <math>\boxed{\dfrac{4}{15}}.</math> | Jordan has <math>10</math> high top sneakers, and <math>6</math> white sneakers. We would want as many white high-top sneakers as possible, so we set <math>6</math> high-top sneakers to be white. Then, we have <math>10-6=4</math> red high-top sneakers, so the answer is <math>\boxed{\dfrac{4}{15}}.</math> |
Revision as of 16:24, 25 January 2024
Problem
Jordan owns pairs of sneakers. Three fifths of the pairs are red and the rest are white. Two thirds of the pairs are high-top and the rest are low-top. The red high-top sneakers make up a fraction of the collection. What is the least possible value of this fraction?
(A) (B) (C) (D) (E)
Solution
Jordan has high top sneakers, and white sneakers. We would want as many white high-top sneakers as possible, so we set high-top sneakers to be white. Then, we have red high-top sneakers, so the answer is